+0  
 
0
563
3
avatar

Altitudes \(\overline{AP}\) and \(\overline{BQ}\) of an acute triangle \(\triangle ABC\) intersect at point \(H\). If \(HP=5\) while HQ=2, then calculate \((BP)(PC)-(AQ)(QC)\) .

 Dec 16, 2018
 #1
avatar+773 
0

\((BP)(PC) - (AQ)(QC) = \boxed{21}\).

 

You are welcome. 

 

- PM

 Dec 17, 2018
 #2
avatar+773 
-1

Check here.

PartialMathematician  Dec 17, 2018
 #3
avatar+773 
-1

Or here.

PartialMathematician  Dec 17, 2018

3 Online Users

avatar